BECOM Test 4

Ace your homework & exams now with Quizwiz!

Oxaloacetate provides C for the synthesis of which AAs?

aspartate and asparagine

Which type of blood flow is characterized by sound production that can be heard w a stethoscope?

turbulent

A patient injects their morning insulin but forgets to eat breakfast and goes into insulin shock. What is the primary cause of this shock?

-Multiple tissues are competing w the CNS for limited amnts of glucose. -insulin will promote the localization of GLUT4 to the cell surface of m. and adipose. This receptor has a high affinity (low Km) for glucose. Since there is no dietary glucose available and minimal stored glucose, these tissues are now competing for glucose w the CNS.

Describe the sickling behavior in red cells of a person w sickle Hb. What causes the cell to sickle? What are irreversibly sickled cells?

-Only the DEOXYGENATED form of HbS is unstable/aggregates -HbS that has reached the lungs is fully oxygenated and adopts the normal oxy conformation. Cells are in normal, raft-like shape -As cells move to periphery, O2 partial pressure drops. HbS becomes deoxygenated. When enough deoxygenated HbS is present, it will begin to aggregate. -Formation of HbS aggregates (fibrous type strugtures) in the cell cause the cell to sickle. Cells become rigid/inflexible, clog capillaries (lead to vascular infarctions) -the ability of cells to rebound (i.e. non-sickle shape when re-oxygenated) is limited. As the sickling cycle continues over and over, many RBCs become irreversibly sickled.

Treatments for sickle cell include hydration, warm compresses, blood transfusions, and nasal oxygen. Why do these help? How does HbS aggregate when O2 is limited but does not at high conc. of O2?

-Since the conc. of Hb in the RBC is very high, small changes in conc. might have marked consequences on the aggregation of the HbSS -Hydration: dehydration promotes crisis -Warm compress: heat increases solubility, treating areas where occlusions are may help solubilize some of the aggregates -Blood transfusions: supply red cells w normal Hb. This may help for a while, but as these normal cells are degraded over their lifetime, the benefit will be lost. --problem: no mechanism for getting rid of excess iron, and iron overload is serious problem -Nasal O2 delivery: potentiallly increases the partial pressure of O2 at the lung and allows for more O2 delivery into the blood system, giving more oxygenated form. -none of these techniques can do a lot but try to do whatever possible to improve in the middle of a crisis -HbS aggregation when O2 is limited, conc. of deoxy form increases -When O2 is abundant, most of the Hb is in the oxy form. -the mutation substitution is obviously not as critical for the solubility of oxy conformation as it is for that of the deoxy conformation -This must mean that the environments around the AA side chains of the Beta-6 position are diff. in the 2 forms

Why does someone w HbS have a higher % of HbF in their red cells than someone w normal HbA?

-common, but why this happens is not clear -prob attempt by the body to maintain some functional oxygen-carrying capacity by producing an Hb that isn't destroyed prematurely by the body --comes at the expense of having a Hb with decreased ability to deliver O2 to tissues, but the tradeoff seems beneficial -Most HbAA individuals may have a few % of HbF. Homozygous HbSS indiv. have as much as 20% -Hydroxyurea therapy increases the amnt of HbF production, and thus delivers this benefit to HbSS patients.

What are the 3 types of capillaries?

-continuous (somatic) -fenestrated (visceral) -sinusoidal (discontinuous)

What factors determine intensity of heart sounds?

-distance separating leaflets at onset of ventricular contraction -mobility of leaflets -rate of rise of ventricular pressure

When would Procrit injections of EPO be indicated?

-in pts w kidney failure where kidney quits producing EPO and pt. has anemia

Describe laminar blood flow.

-layered flow -blood flows in concentric layers -each layer of blood flows at a diff velocity -no sound production that can be heard w a stethoscope

What types of things could exacerbate a sickle cell crisis?

-low blood pH -high levels of 2,3-BPG

How are the nonessential AAs synthesized by the body?

-made from glucose and an N source (another AA or NH3) -Tyrosine and cysteine require an essential AA for synthesis --phenylalanine needed for Tyr --methionine needed for cysteine

What are the important enzymes for amino acid metabolism?

-pyridoxal phosphate -tetrahydrofolate (FH4) -tetrahydrobiopetrin (BH4)

Explain heterozygosity for sickle cell. What is the selection advantage?

-the heterozygosity leads to the HbAS trait - inherit one Hb gene from each parent, if one allele is HbA type and one is HbS type, the person possesses both kinds of Beta-subunits -If both parents have the trait HbAS, there is a 25% chance for each of their children to be homozygous HbSS (or HbAA) -Selection advantage - related to the resistance to the full life cycle of the malaria parasite in those w HbAS. Concentrate the sickle gene among those who are in malaria-prone parts of the world. Possible mechanisms: --parasite significantly reduces the O2 tension of infected RBCs, causing sickling of that particular RBC - which signals phagocytes to get rid of that RBC (and the parasite w/in). Increase in phagocytic activity in indiv. w sickle cell trait. Since the sickling of parasite infected cells is higher, these selectively get removed, sparing the normal erythrocytes --Excessive vacuole formation occurs in those parasites infecting sickle cells --Sickle trait erythrocytes produce higher levels of the superoxide anion and hydrogen peroxide than normal erythrocytes, both are toxic to parasites

What are the 3 layers of a blood vessel?

-tunica intima -tunica media -tunica adventitia (externa)

What are the factors that govern blood flow?

-viscosity -length -radius -pressure gradients

10. A 76-year-old man with a history of prostatic hypertrophy has the recent onset of increased difficulty urinating. Symptoms began shortly after he started taking a nasal decongestant orally for cold symptoms. Which of the following types of receptors is most likely to be involved in these adverse effects? (A) α1-Adrenergic (B) β2-Adrenergic (C) Ganglionic nicotinic (D) Nicotinic receptor at the neuromuscular junction (E) Serotoninergic

A

100. A 28-year-old woman comes to the physician because of a 6-month history of intermittent feelings of progressive overwhelming fear and apprehension, restlessness, and palpitations. She says that she often feels tense, and she has difficulty concentrating because she is unable to stop thinking about the things that worry her. Physical examination shows no abnormalities. A selective serotonin reuptake inhibitor (SSRI) is prescribed for long-term management. Which of the following is most appropriate to provide immediate relief for this patient until the effects of the SSRI treatment occur? (A) Alprazolam (B) Amitriptyline (C) Diphenhydramine (D) Haloperidol (E) Phenobarbital (F) Tramadol

A

105. A 37-year-old woman comes to the physician because of a 2-month history of pain with movement of her hands and feet. Physical examination shows warmth and swelling of the metacarpophalangeal and metatarsophalangeal joints. Laboratory studies show increased titers of antibodies to Fc component of IgG and a negative antinuclear antibody test result. A drug is prescribed that binds to tumor necrosis factor- α (TNF-α) and blocks its interaction with cellsurface TNF receptors. Her symptoms improve within 1 month. She is most likely receiving treatment with which of the following drugs? (A) Adalimumab (B) Anakinra (C) Gold (D) Methotrexate (E) Prednisone

A

109. A 36-year-old woman has been trying to conceive for the past 2 years. Her menses occur every 19 to 45 days. She has a past history of some type of sexually transmitted disease, but says she was treated and cured. She is 163 cm (5 ft 4 in) tall and weighs 109 kg (240 lb); BMI is 41 kg/m2 . Physical examination including a pelvic examination is unremarkable. An endometrial biopsy is performed based on the menstrual history and a negative pregnancy test. The biopsy shows stromal breakdown associated with proliferative glands. Which of the following is the most likely cause of her infertility? (A) Anovulation (B) Chronic endometritis (C) Endometrial polyps (D) Endometriosis (E) Leiomyomata

A

110. A 1-day-old newborn is evaluated for possible sepsis. Blood cultures grow gram-positive cocci in pairs and chains that agglutinate with group B antiserum. The most likely epidemiologic risk factor for this infection involves bacterial colonization of which of the following? (A) Mother's vagina (B) Newborn's gastrointestinal tract (C) Newborn's nasopharynx (D) Placenta (E) Umbilical cord remnant

A

112. A 29-year-old man is brought to the physician for removal of a cast from his left leg. He sustained a fracture of the left lower extremity 6 weeks ago and was immobilized in a cast that extended from just below the knee to the foot. At the time of injury, there was severe pain but normal strength in the extremity. When the cast is removed today, physical examination shows a pronounced left footdrop with paresthesia and sensory loss over the dorsum of the left foot and lateral leg. Injury to which of the following nerves is the most likely cause of this patient's condition? (A) Common fibular (peroneal) (B) Femoral (C) Obturator (D) Sciatic (E) Tibial

A

117. An investigator is conducting a study of hypertension in an experimental animal model. Results show that hypertension is induced when one renal artery is constricted with a clamp, but both kidneys remain intact. Which of the following best explains the contribution of the normal kidney to the hypertension in this experimental model? (A) Aldosterone-induced sodium and water retention (B) Angiotensin I-induced vasoconstriction (C) Angiotensin II-induced renal vein constriction (D) Expression of angiotensin-converting enzyme (E) Increased renin secretion

A

119. A 68-year-old man comes to the physician because of worsening nonproductive cough and progressive shortness of breath during the past 2 months. Three months ago, he was able to walk an unlimited distance without difficulty; now, he becomes short of breath after walking one block. He has had no fever, chest pain, or leg swelling. He has atrial fibrillation and hypertension treated with medications. His temperature is 37.7°C (99.8°F), pulse is 90/min, and respirations are 22/min. Pulse oximetry on room air shows an oxygen saturation of 92%. Diffuse inspiratory crackles are heard bilaterally. The remainder of the examination shows no abnormalities. His plasma brain natriuretic peptide concentration is within the reference range. A chest x-ray shows bilateral interstitial infiltrates. Which of the following medications is the most likely cause of these findings? (A) Amiodarone (B) Digoxin (C) Lisinopril (D) Metoprolol (E) Procainamide

A

121. A 52-year-old woman comes to the physician because of a 1-month history of headache, weakness, tingling of her extremities, muscle cramping, and fatigue. Her blood pressure is 170/110 mm Hg. Physical examination shows no other abnormalities. Laboratory studies show a decreased serum potassium concentration, metabolic alkalosis, and decreased plasma renin activity; serum sodium concentration is within the reference range. Urine catecholamine concentrations are within the reference range. Which of the following is the most likely diagnosis? (A) Adrenal adenoma (B) Focal segmental glomerulosclerosis (C) Hypothalamic tumor (D) Juxtaglomerular cell tumor (E) Renal artery stenosis

A

36. A 35-year-old man who works at a facility processing highly radioactive substances accidentally receives a high, whole-body dose of ionizing radiation estimated to be 1500 rads (15 gray). He dies 1 week later. At autopsy, histologic examination of the skin shows scattered, individual epidermal cells with shrunken, markedly eosinophilic cytoplasm and pyknotic, fragmented nuclei. These morphologic changes most likely indicate which of the following processes? (A) Apoptosis (B) Coagulation necrosis (C) Liquefaction necrosis (D) Mutagenesis (E) Tumor initiation

A

38. A 45-year-old man comes to the physician because of right shoulder pain that began after he chopped wood 2 days ago. Examination of the right upper extremity shows no obvious bone deformities or point tenderness. The pain is reproduced when the patient is asked to externally rotate the shoulder against resistance; there is no weakness. In addition to the teres minor, inflammation of which of the following tendons is most likely in this patient? (A) Infraspinatus (B) Pectoralis (C) Subscapularis (D) Supraspinatus (E) Trapezius

A

42. A full-term female newborn is examined shortly after birth. She appears to be small for gestational age, and she has excess skin on the nape of the neck and lymphedema of the hands and feet. Chromosomal analysis shows some cells with a normal 46,XY karyotype and some cells with a 45,X karyotype. Which of the following mechanisms best explains this cytogenetic abnormality? (A) Nondisjunction in mitosis (B) Reciprocal translocation (C) Robertsonian translocation (D) Skewed X-inactivation (E) Uniparental disomy

A

55. A 15-year-old girl who is a ballet dancer has not had a menstrual period for the past 3 months. Menses were previously regular at 29-day intervals. She has lost weight over the past year; her weight is 70% of that expected for her height. She is afebrile and has purpuric lesions on her extremities and trunk. Platelet, absolute neutrophil, and lymphocyte counts are below the reference range. She has macrocytic anemia. The most likely cause of these symptoms is a deficiency of which of the following nutrients? (A) Folic acid (B) Iron (C) Linoleic acid (D) Magnesium (E) Niacin (F) Protein (G) Vitamin A (H) Vitamin B6 (pyridoxine) (I) Vitamin C (J) Vitamin D (K) Vitamin E (L) Vitamin K (M) Zinc

A

65. Six healthy subjects participate in a study of muscle metabolism during which hyperglycemia and hyperinsulinemia is induced. Muscle biopsy specimens obtained from the subjects during the resting state show significantly increased concentrations of malonyl-CoA. The increased malonyl-CoA concentration most likely directly inhibits which of the following processes in these subjects? (A) Fatty acid oxidation (B) Fatty acid synthesis (C) Gluconeogenesis (D) Glycogenolysis (E) Glycolysis (F) Oxidative phosphorylation

A

80. A 55-year-old man comes to the physician because of a 2-week history of recurrent, widespread blister formation. Physical examination shows lesions that are most numerous in the flexural areas including the axillae and groin. The blisters do not break easily, and there are no oral lesions. These blisters are most likely the result of adhesion failure involving which of the following? (A) Basement membrane (B) Dermal papillae (C) Langerhans cells (D) Melanocytes (E) Merkel cells

A

82. A 16-year-old boy is admitted to the emergency department because of a knife wound to the left side of his chest. An x-ray of the chest shows an air-fluid level in the left side of the chest, partial collapse of the left lung, and elevation of the stomach bubble. The mediastinum is in the midline. Which of the following is the most likely diagnosis? (A) Hemopneumothorax, not under tension (B) Hemothorax, not under tension (C) Pneumothorax, not under tension (D) Tension hemopneumothorax (E) Tension hemothorax (F) Tension pneumothorax

A

92. After being severely beaten and sustaining a gunshot wound to the abdomen, a 42-year-old woman undergoes resection of a perforated small bowel. During the operation, plastic reconstruction of facial fractures, and open reduction and internal fixation of the left femur are also done. Thirty-six hours postoperatively, she is awake but not completely alert. She is receiving intravenous morphine via a patientcontrolled pump. She says that she needs the morphine to treat her pain, but she is worried that she is becoming addicted. She has no history of substance use disorder. She drinks one to two glasses of wine weekly. Which of the following initial actions by the physician is most appropriate? (A) Reassure the patient that her chance of becoming addicted to narcotics is minuscule (B) Maintain the morphine, but periodically administer intravenous naloxone (C) Switch the patient to oral acetaminophen as soon as she can take medication orally (D) Switch the patient to intramuscular lorazepam (E) Switch the patient to intravenous phenobarbital

A

If an individual has a vit B6 deficiency, which of the following AAs could still be synthesized and be considered nonessential? A. Tyrosine B. Serine C. Alanine D. Cysteine E. Aspartate

A. Tyrosine is derived from phenylalanine, which requires BH4 but not vit B6. Vit. B6 is required in the synthesis of serine (transamination), alanine (another transamination), cysteine (beta-elimination, beta-addition, beta-elimination), and aspartate (transamination)

111. A 26-year-old woman is brought to the emergency department by her mother 1 hour after she had a generalized tonic-clonic seizure at home. The mother states that her daughter has been talking to herself at all hours of the day and night about being thirsty. The patient was found to have schizophrenia 6 years ago. Her only medication is haloperidol. She was admitted to the hospital twice in the past year for psychotic episodes. Physical examination shows no other abnormalities. She is oriented to person but not to place or time. Her serum sodium concentration is 114 mEq/L on arrival but returns to normal with appropriate treatment. Which of the following interventions is the most appropriate next step? (A) Refer the patient for behavior therapy (B) Restrict the patient's fluid intake (C) Add lithium carbonate to the medication regimen (D) Administer furosemide therapy (E) Begin mineralocorticoid therapy

B

123. An 84-year-old woman who resides in an assisted living facility is brought to the emergency department because of fever and cough for 1 week. The cough has been productive of foul-smelling, yellow-green sputum for 24 hours. She has a 2-year history of dementia, Alzheimer type. Her temperature is 38.5°C (101.3°F), pulse is 80/min, respirations are 20/min, and blood pressure is 116/66 mm Hg. Coarse inspiratory crackles are heard over the right lung field. Laboratory studies show a leukocyte count of 13,500/mm3 (72% segmented neutrophils, 8% bands, 1% eosinophils, 16% lymphocytes, and 3% monocytes). A CT scan shows a cavitary lesion in the superior segment of the right lower lobe. The lesion has a thick wall and an irregular peripheral margin; there is no displacement of the adjacent bronchovascular bundle. Which of the following is the most likely cause of the lung lesion in this patient? (A) Antecedent viral pneumonia (B) Aspiration of gastric contents (C) Bronchial obstruction by metastatic carcinoma (D) Lung infarction secondary to arterial thrombosis (E) Primary carcinoma of the lung (F) Secondary infection of a congenital lung cyst (G) Septic embolism from an extrapulmonary site

B

127. A 37-year-old woman with right lower extremity edema is evaluated because of the sudden onset of shortness of breath and pleuritic chest pain. A diagnosis of pulmonary embolism is made. Which of the following signs, if present on physical examination, would be the most specific indicator of pulmonary arterial hypertension in this patient? (A) Increased jugular venous pressure (B) P2 louder than A2 (C) Peripheral edema (D) Presence of an S3 (E) Pulmonary crackles

B

131. A 55-year-old man with a history of drug and alcohol abuse undergoes operative placement of a portosystemic shunt to relieve portal hypertension. During this procedure, it is most appropriate for the physician to anastomose a major tributary of the portal vein to which of the following vessels? (A) Left gastric vein (B) Left renal vein (C) Splenic vein (D) Superior mesenteric vein (E) Umbilical vein

B

19. Vasovagal syncope is partially caused by increased: A. sympathetic nerve activity to the heart B. parasympathetic nerve activity to the heart C. brain blood flow D. parasympathetic nerve activity to systemic arterioles E. sympathetic nerve activity to systemic arterioles

B

47. A 70-year-old man is brought to the emergency department by his wife because of fever and shortness of breath for 2 days. He underwent an oral surgical procedure 6 weeks ago. His respirations are 22/min, and blood pressure is 140/60 mm Hg. A soft diastolic murmur is heard. The diagnosis of bacterial endocarditis is made. Gentamicin therapy is initiated. This patient is at increased risk for developing which of the following as a result of this therapy? (A) Cardiac ischemia (B) Hearing loss (C) Hyperglycemia (D) Lung infection (E) Torsades de pointes

B

6. Which of the following sequences is correct? A. Ð Contractility Ð End systolic volume Ð Stroke volume B. Ð Contractility ø End systolic volume Ð Stroke volume C. Ð Contractility Ð End systolic volume ø Stroke volume D. ø Contractility ø End systolic volume ø Stroke volume E. ø Contractility Ð End systolic volume Ð Stroke volume

B

67. A 23-year-old woman has a progressive increase in her serum β-human chorionic gonadotropin (β-hCG) concentrations during an 8-week period. A hydatidiform mole is removed, but the β-hCG concentration continues to increase. Which of the following is the most likely diagnosis? (A) Adrenal adenoma (B) Choriocarcinoma (C) Ectopic pregnancy (D) Pituitary insufficiency (E) A second noninvasive mole

B

69. A 75-year-old woman has increasing shortness of breath on exertion. Findings on physical examination are unremarkable. X-rays of the chest show no abnormalities of the heart or lungs. Pertinent laboratory findings include: Hematocrit 28% Hemoglobin 9 g/dL Mean corpuscular volume 70 μm3 Which of the following is the most likely basis for these findings? (A) Acquired hemolytic anemia (B) Chronic blood loss (C) Folic acid deficiency (D) β-Thalassemia minor (E) Pernicious anemia

B

81. A study is designed to evaluate the feasibility of acupuncture in children with chronic headaches. Sixty children with chronic headaches are recruited for the study. In addition to their usual therapy, all children are treated with acupuncture three times a week for 2 months. Which of the following best describes this study design? (A) Case-control (B) Case series (C) Crossover (D) Cross-sectional (E) Historical cohort (F) Randomized clinical trial

B

84. A 74-year-old man with urinary frequency and urgency has benign prostatic hyperplasia. He refuses operative intervention but agrees to a trial of finasteride therapy. During the trial, synthesis of which of the following substances is most likely to be inhibited? (A) Androstenedione (B) Dihydrotestosterone (C) Estradiol (D) Estrone (E) Testosterone

B

85. A 30-year-old man with peptic ulcer disease suddenly develops pain, redness, and swelling of his right first metatarsophalangeal joint. There is no history of injury. Serum uric acid concentration is 8 mg/dL. Examination of joint aspirate shows birefringent crystals. Which of the following drugs is most appropriate to treat the acute symptoms in this patient? (A) Allopurinol (B) Colchicine (C) Morphine (D) Probenecid (E) Sulfinpyrazone

B

95. A 6-day-old breast-fed boy is brought to the emergency department by his mother because of poor weight gain and irritability since delivery, and a 2-hour history of vomiting. Physical examination shows jaundice and hepatomegaly. A reducing substance test result of the urine is positive, and a glucose oxidase test result is negative. The concentration of which of the following metabolites in liver is most likely increased in this patient? (A) Fructose 1,6-bisphosphate (B) Galactose 1-phosphate (C) Glucose 1-phosphate (D) Glucose 6-phosphate

B

Which of the forms of FH4 is required for the synthesis of methionine from homocysteine?

B. The only 3 forms of folate that transfer Cs are the N5-methyl-FH4 form, the N5,N10-methylene FH4 form, and the N10-formyl form. None of the other forms participates in rxns in which the C is transferred. It is the N5-methyl form that transfers the methyl group to form methionine from homocysteine.

101. A healthy 28-year-old woman comes to the physician for advice on losing weight. She is 150 cm (4 ft 11 in) tall and weighs 56 kg (124 lb); BMI is 25 kg/m2 . Physical examination shows no other abnormalities. The physician recommends a diet that will restrict her daily intake by 500 kilocalories. Which of the following processes is most likely to increase in this patient as a result of following this diet? (A) Adipocyte glucose uptake (B) Cerebral ketone utilization (C) Hepatic lipid oxidation (D) Muscle glucose uptake (E) Resting energy expenditure

C

115. A 10-year-old boy is brought to a new physician by his parents for an initial examination. The patient was born with congenital glaucoma, hearing loss, and a patent ductus arteriosus that has since been surgically corrected. At the time of birth, he also had purpura, jaundice, and splenomegaly. He is in a special education class for students with mental retardation. He is at the 75th percentile for height and weight, and 5th percentile for head circumference. This patient's condition was most likely caused by in utero exposure to which of the following? (A) Cytomegalovirus infection (B) HIV infection (C) Rubella (D) Syphilis (E) Toxoplasmosis

C

132. A 52-year-old woman is admitted to the hospital because of breast cancer metastatic to the liver. Her prognosis is poor. She begs her husband to stay with her at the hospital because she is afraid to be left alone. Which of the following defense mechanisms best explains her behavior? (A) Denial (B) Displacement (C) Regression (D) Repression (E) Sublimation

C

45. A 37-year-old woman has blurred, double vision 8 hours after eating home-preserved peppers. Six hours later, she has dysphagia, dry mouth and eyes, progressive weakness of the arms and legs, and urinary retention. She is awake and alert. Which of the following is the most likely mechanism of these adverse effects? (A) Antagonism of muscarinic receptors (B) Antagonism of nicotinic receptors (C) Inhibition of acetylcholine release (D) Inhibition of cholinesterase activity (E) Inhibition of G proteins

C

49. A 31-year-old woman comes to the physician because of a 2-week history of malaise, nausea, vomiting, and decreased appetite. She is a known user of intravenous heroin. She appears chronically ill. She is 165 cm (5 ft 5 in) tall and weighs 47 kg (103 lb); BMI is 17 kg/m2 . Her temperature is 36.7°C (98.1°F), pulse is 90/min, respirations are 18/min, and blood pressure is 114/68 mm Hg. Physical examination shows scleral icterus and a liver span of 16 cm. The spleen is not palpable. Serum studies show: Total bilirubin 3.2 mg/dL AST 774 U/L ALT 820 U/L HIV antibody negative Hepatitis B surface antigen negative Hepatitis B surface antibody positive Anti-hepatitis B core antibody positive Hepatitis B DNA negative Anti-hepatitis C virus positive Hepatitis C RNA positive Which of the following is the most likely outcome of this patient's infection? (A) Complete resolution of infection (B) Latent infection with intermittent viremia (C) Lifelong persistent infection (D) Patient death from acute infection

C

62. Over 1 year, a study is conducted to assess the antileukemic activity of a new tyrosine kinase inhibitor in patients with chronic myeloid leukemia in blast crisis. All patients enrolled in the study are informed that they would be treated with the tyrosine kinase inhibitor. They are assigned to successive dose cohorts of 300 to 1000 mg/day of the drug. Six to eight patients are assigned to each dose. Treatment efficacy is determined based on the results of complete blood counts and bone marrow assessments conducted regularly throughout the study. This study is best described as which of the following? (A) Case-control study (B) Crossover study (C) Open-labeled clinical trial (D) Randomized clinical trial (E) Single-blind, randomized, controlled trial

C

63. A previously healthy 19-year-old woman comes to the physician because of a 3-day history of fever, fatigue, and sore throat. She lives with a roommate who has a cat. Her temperature is 37.8°C (100°F). Physical examination shows mildly tender cervical and submental adenopathy and pharyngitis. There is mild splenomegaly. Laboratory studies show: Hemoglobin 13.3 g/dL Hematocrit 43% Leukocyte count 12,500/mm3 Platelet count 250,000/mm3 Heterophile antibody titer positive The most likely cause of this patient's condition is infection with which of the following? (A) Bartonella henselae (B) Cytomegalovirus (C) Epstein-Barr virus (D) Rhinovirus (E) Toxoplasma gondii

C

64. A previously healthy 3-month-old boy is brought to the physician because of a runny nose and a dry cough for 2 days. Physical examination shows tachypnea, a nasal discharge, and wheezing. An x-ray of the chest shows hyperexpansion but no infiltrates. The causal virus was most likely transmitted by which of the following routes? (A) Blood transfusion (B) Ingestion of contaminated formula (C) Inoculation onto mucous membranes (D) Insect bite (E) Transplacental transfer

C

70. A 42-year-old woman is brought to the emergency department 4 hours after the onset of severe shortness of breath. She has no recent history of trauma, hospital admission, or operations. She had an episode of deep venous thrombosis 10 years ago that required treatment in the hospital. Her respirations are 34/min. Pulse oximetry on room air shows an oxygen saturation of 65%. A helical CT scan shows a large filling defect in the right pulmonary artery. Which of the following hypercoagulability disorders is the most likely underlying cause of these findings? (A) Antiplatelet antibody syndrome (B) Antithrombin III deficiency (C) Factor V Leiden mutation (D) Protein C deficiency (E) Protein S deficiency

C

73. A 12-year-old boy is brought to the physician by his father because of redness and swelling of his left foot for 24 hours. Three days ago, the boy scraped his foot while wading in a drainage ditch. Examination of the left foot shows a purulent abrasion with edema, erythema, and tenderness on the lateral side. Infection is most likely to next spread from the lateral side of the foot to the regional lymph nodes in which of the following areas? (A) Lateral surface of the thigh (B) Medial malleolus, posteriorly (C) Popliteal fossa (D) Sole of the foot (E) Superficial inguinal area

C

78. A 32-year-old man with non-Hodgkin lymphoma comes to the physician 6 days after finishing the initial chemotherapy regimen. His leukocyte count is 1600/mm3 , indicating greater bone marrow suppression than expected. When questioned, the patient says that he has been taking Madagascar periwinkle as an herbal remedy for his condition. He obtains this substance from an herbalist. Which of the following is the most appropriate response by the physician? (A) Ask the patient to stop using the herbal supplement because supplements are generally ineffective (B) Continue the patient's chemotherapy (C) Explain the adverse effects this herbal supplement has on the patient's treatment (D) Report the herbalist to the Food and Drug Administration (E) Suggest that the patient take daily multivitamin and protein supplements in addition to the herbal supplement

C

How is AA synthesis regulated?

feedback regulation -as conc. of free AAs increase, key enzymes are allosterically or transcriptionally inhibited -AA levels are maintained at the level where aminoacyl-tRNA synthetases are still active so protein synthesis continues

What type of capillaries are found where rapid exchange is required (kidney, intestines, endocrine glands, choroid plexus)?

fenestrated (visceral)

Where are the carbon skeletons for the 10 nonessential AAs derived from?

glucose

Alpha ketoglutarate is a precursor for the synthesis of which AAs?

glutamate, glutamine, proline, arginine

What is needed for porphyrin synthesis?

glycine, succinyl coA, vit. B6

124. A 20-year-old man comes to the physician's office for a scheduled health maintenance examination. His father died of a myocardial infarction at age 55 years. Physical examination shows a tendon xanthoma on the elbow. His serum total cholesterol concentration is 360 mg/dL. A mutation is most likely to be found in which of the following genes? (A) apoA2 (B) apoC2 (C) apoE-ε4 (D) LDL receptor (E) VLDL receptor

D

21. A 28-year-old man comes to the physician because of a 1-year history of pain with urination that has increased in severity during the past month. He also has had episodes of blood in his urine during the past 5 years. He lived in subSaharan Africa until he came to the USA 6 months ago for graduate school. His temperature is 38°C (100.4°F), pulse is 80/min, respirations are 16/min, and blood pressure is 110/84 mm Hg. Physical examination shows suprapubic tenderness. Laboratory studies show: Hemoglobin 12.3 g/dL Hematocrit 37% Leukocyte count 13,400/mm3 Segmented neutrophils 65% Bands 5% Eosinophils 5% Lymphocytes 22% Monocytes 3% Serum Urea nitrogen 75 mg/dL Creatinine 3.8 mg/dL Urine Blood 3+ RBC 200/hpf WBC 100/hpf RBC casts absent WBC casts absent Imaging studies show bilateral hydroureter and hydronephrosis and foci of calcification in the region of the bladder. A biopsy specimen of the bladder shows marked chronic inflammation with fibrosis and scattered granulomas. Which of the following best explains the biopsy findings? (A) Exposure to a chemical toxin (B) Interstitial cystitis (C) Malacoplakia (D) Schistosomiasis (E) Vesicoureteral reflux

D

3. The pacemaker potential of sinoatrial (SA) nodal cells: A. reaches threshold more slowly than the pacemaker potential of atrioventricular (AV) nodal cells B. is primarily the result of opening of channels carrying K+ ions C. rises to threshold more rapidly in the presence of a beta adrenergic antagonist D. rises to threshold more slowly in the presence of acetylcholine E. A and C are correct.

D

54. A patient being treated with clindamycin for aspiration pneumonia develops diarrhea. The stool contains a toxin that kills cultured epithelial cells. Stool culture grows an anaerobic gram-positive rod. The same organism is cultured from his bedpan. Which of the following is most likely to sterilize the bedpan? (A) Boiling for 45 minutes (B) Exposure to benzalkonium chloride for 1 hour (C) Exposure to ethyl alcohol for 1 hour (D) Exposure to saturated steam (121°C) for 15 minutes (E) Heating in an oven at 150°C for 30 minutes

D

56. An otherwise healthy 26-year-old woman has had petechiae on her legs during the last 24 hours. Laboratory studies show: Hemoglobin 13.1 g/dL Hematocrit 39.7% Leukocyte count 8500/mm3 Neutrophils 65% Lymphocytes 30% Monocytes 5% Mean corpuscular volume 82.2 μm3 Platelet count 20,000/mm3 A peripheral blood smear shows normal red cell morphology; a bone marrow smear shows mature megakaryocytic hyperplasia. Which of the following is the most likely diagnosis? (A) Acute megakaryocytic leukemia (B) Acute myelogenous leukemia (C) Aplastic anemia (D) Immune thrombocytopenic purpura (E) Epstein-Barr viral infection (F) Papovavirus infection (G) Thrombotic thrombocytopenic purpura

D

59. A 66-year-old man is brought to the emergency department by neighbors 1 hour after the sudden onset of progressive confusion and sleepiness. He had just seen the physician 1 week before, and he appeared cheerful and his usual sharp self until he was found wandering in his neighbor's yard the day of admission. He appears sleepy, but he is arousable. He is oriented to person, but not to place or time. He recalls two of three objects after 5 minutes. He says, "There are so many little people here. What sort of place is this?" He then falls back asleep. Which of the following is the most likely cause of this patient's condition? (A) Acute paranoid schizophrenia (B) Arsenic poisoning (C) Dementia, Alzheimer type (D) New medication regimen (E) Tertiary syphilis

D

66. A 6-year-old boy is brought to the physician by his parents because of a 3-day history of fever, headache, and cough productive of a green, foulsmelling discharge that also exits from his nose. He has had repeated episodes of similar symptoms during the past 4 years. He appears pale and lethargic. His height and weight are both below the 10th percentile. Coarse rhonchi are heard bilaterally. An x-ray of the chest shows scattered peripheral opacities, dilated and thickened airways consistent with bronchiectasis, and a cardiac apex that is directed toward the right. The most likely cause of his recurrent infections is a dysfunction of which of the following cell types? (A) Alveolar capillary endothelial cell (B) Alveolar macrophage (C) Chondrocyte (D) Ciliated columnar epithelial cell (E) Clara cell (F) Goblet cell (G) Kulchitsky cell (H) Squamous epithelial cell (I) Type I pneumocyte (J) Type II pneumocyte

D

68. A 45-year-old man has abnormal circadian variation in body temperature, disruption of the sleep-wake cycle, and an impaired nocturnal surge of secretion of melatonin. An MRI of the brain is most likely to show a lesion involving which of the following nuclei? (A) Accessory optic (B) Lateral preoptic (C) Pretectal (D) Suprachiasmatic (E) Supraoptic

D

71. A 12-year-old girl is admitted to the hospital because of marked shortness of breath, an erythematous rash, and painful, swollen hip and knee joints. She is agitated. A chest x-ray shows an enlarged heart and changes consistent with pulmonary edema. Intractable congestive heart failure develops, and she dies on the second hospital day. This child most likely had a recent history of which of the following? (A) Cyanosis with chest pain (B) Jaundice (C) Meningitis (D) Pharyngitis (E) Skin infection

D

72. A 5-year-old child with short stature is being evaluated for delayed dentition and excessive caries. Examination of the chest shows pectus carinatum and bead-like enlargement of the costochondral junctions. Which of the following findings is most likely on histologic examination of a section of bone? (A) Absence of cartilage in the epiphyseal plates (B) Absence of osteoblasts (C) Enlarged osteoclasts with an increased number of nuclei (D) Increased proportions of osteoid

D

77. In a cohort study of elderly women, the relative risk ratio for hip fractures among those who exercise regularly is 1.2 (95% confidence interval of 1.1 to 1.8). Which of the following is the most appropriate conclusion about the effect of regular exercise on the risk for hip fracture? (A) Statistically nonsignificant increase in risk (B) Statistically nonsignificant overall decrease in risk (C) Statistically significant overall decrease in risk (D) Statistically significant overall increase in risk

D

8. Left ventricular afterload can be increased by: A. increasing aortic pressure B. increasing left ventricular systolic pressure C. decreasing left ventricular end-diastolic radius D. Only A and B are correct. E. A, B, and C are correct.

D

88. A previously healthy 40-year-old man is brought to the emergency department because of constant substernal chest pain for 12 hours that is exacerbated by coughing and inspiration. The pain is relieved with sitting up and leaning forward. There is no family history of heart disease. His temperature is 38°C (100.4°F), pulse is 120/min, and blood pressure is 110/60 mm Hg. The lungs are clear to auscultation. Cardiac examination shows distant heart sounds. An ECG shows diffuse ST-segment elevation in all leads. An x-ray of the chest shows normal findings. The most likely cause of his condition is injury to which of the following tissues? (A) Aortic intima (B) Esophageal sphincter (C) Myocardium (D) Pericardium (E) Pleura

D

90. A 22-year-old man comes to the emergency department because of the recent onset of torticollis and uncontrollable facial grimacing. He began therapy with a new drug 24 hours ago. Which of the following drugs is the most likely cause? (A) Amitriptyline (B) Diazepam (C) Fluoxetine (D) Haloperidol (E) Levodopa

D

What type of treatment is indicated in patients with porphyrias?

high carb diet

Pt's w sickle cell disease can be treated w hydroxyurea to:

increase gamma-globin expression

Compared to adult hemoglobin, the oxygen affinity of fetal globin is:

it depends on local [2,3-BPG]

If a person begins hyperventilating what is likely to happen to their blood pH?

it will increase

What is one of the major complications of untreated jaundice?

kernicterus

AAs whose C skeletons can be converted directly to acetyl coA or acetoacetate are called what?

ketogenic

Which organ is the main site of production of EPO?

kidneys

102. A 25-year-old woman has a routine, pre-employment physical examination. Laboratory studies include: Hemoglobin 11.3 g/dL Hematocrit 34% Erythrocyte count 5.2 million/mm3 Mean corpuscular volume 65 μm3 Follow-up laboratory studies show that the serum iron concentration and iron-binding capacity are within the reference ranges. Hemoglobin electrophoresis shows increased hemoglobin A2 (5%). Which of the following is the most likely diagnosis? (A) Anemia of chronic disease (B) Iron deficiency anemia (C) Sideroblastic anemia (D) α-Thalassemia minor (E) β-Thalassemia minor

E

103. An 18-year-old woman comes to the physician for a health maintenance examination. She has not had major medical illnesses. She takes no medications. She does not smoke cigarettes, drink alcohol, or use illicit drugs. Physical examination shows diffuse brownish yellow discoloration of all teeth. Which of the following most likely occurred during childhood to cause this finding? (A) Amelogenesis imperfecta (B) Dentinogenesis imperfecta (C) Rh incompatibility (D) Syphilis (E) Tetracycline use (F) Vitamin D deficiency

E

108. A 30-year-old woman, gravida 3, para 3, comes to the physician because of a 2-month history of loss of urine when she sneezes or laughs. Her three children were all delivered vaginally at term. Physical examination shows no abnormalities. Which of the following nerve roots supplies the muscle group that is weakened in this patient? (A) C1-2 (B) L1-2 (C) L4-5 (D) S1-2 (E) S3-4

E

114. A 24-year-old woman comes to the physician for a follow-up examination. One week ago, she was treated in the emergency department after she accidentally spilled hot grease on her left leg while working at a fast-food restaurant. Examination of the left lower extremity shows a 7-cm, pink, soft, granular, edematous wound. The formation of this tissue was most likely caused by increased activity of which of the following? (A) Complement C3b (B) Glycosylation-dependent cell adhesion molecule-1 (C) P-selectin (D) Stromelysin (E) Vascular endothelial growth factor

E

118. One day after a 10-km race, a previously healthy 42-year-old man has dark urine. Urinalysis shows: Specific gravity 1.010 Dipstick Glucose negative Blood positive Nitrites negative Microscopic examination WBC negative RBC negative Which of the following is the most likely cause of these findings? (A) Acute glomerulonephritis (B) Hypovolemia (C) Renal infarct (D) Renal vein thrombosis (E) Rhabdomyolysis

E

120. A 2-week-old female newborn delivered at term is brought to the physician by her mother because of an increasingly severe diaper rash since birth. No congenital anomalies were noted after delivery. Physical examination shows a red and swollen umbilical remnant that has not separated. There are ulcerations of the skin but no purulent exudate in the area of the diaper. A culture of one of the ulcers grows Staphylococcus aureus. Despite antibiotic therapy, 1 month later she develops a perirectal fissure, culture of which grows Escherichia coli but a smear of which shows scarce segmented neutrophils. Laboratory studies now show: Hemoglobin 12.7 g/dL Hematocrit 38% Mean corpuscular volume 98 μm3 Leukocyte count 89,790/mm3 Segmented neutrophils 89% Bands 6% Lymphocytes 3% Monocytes 2% Platelet count 249,000/mm3 Serum IgA 92 mg/dL IgG 766 mg/dL IgM 101 mg/dL A peripheral blood smear shows normochromic, normocytic erythrocytes and leukocytes with normal morphology. This patient most likely has which of the following conditions? (A) Acute myelogenous leukemia (B) AIDS (C) Chédiak-Higashi syndrome (D) Common variable immunodeficiency (E) Leukocyte adhesion deficiency

E

122. A 72-year-old man who is a retired construction worker comes to the physician because he has had a lesion on his face for 3 months. Physical examination shows a 6-mm, red, ulcerated lesion with heaped borders. A biopsy specimen of the lesion shows atypical, dysplastic keratinocytes within the epidermis and dermis. Which of the following is the most likely diagnosis? (A) Actinic keratosis (B) Discoid lupus erythematosus (C) Melanoma (D) Mycosis fungoides (E) Squamous cell carcinoma

E

125. A study is conducted to assess the relationship between serum protein YY (PYY) concentrations and obesity. A total of 24 subjects with BMIs ranging from 17 to 40 kg/m2 are enrolled in the study. A fasting serum PYY concentration is determined for each patient. The investigators report that the correlation coefficient between the two parameters is -0.84 (p<0.001). Based on these results, which of the following is the most accurate conclusion? (A) A decreased serum PYY concentration is a cause of obesity (B) The likelihood of a correlation between high BMIs and serum PYY concentrations is 0.16 (C) Obesity is not related to serum PYY concentrations (D) The slope of the line showing the relationship between serum PYY concentrations and BMIs is -0.84 (E) Subjects with lower BMIs have higher serum PYY concentrations

E

128. A 56-year-old man comes to the emergency department because of a 4-day history of colicky right flank pain that radiates to the groin and hematuria. Ultrasound examination of the kidneys shows right-sided hydronephrosis and a dilated ureter. Which of the following is most likely to be found on urinalysis? (A) Erythrocyte casts (B) Glucose (C) Leukocyte casts (D) Oval fat bodies (E) Uric acid crystals

E

14. With lymphatic obstruction, the only safety factor against edema is: A. increased capillary hydrostatic pressure B. increased clearance of plasma protein from interstitial fluid C. increased interstitial fluid protein concentration D. increased plasma protein concentration E. increased interstitial fluid pressure

E

18. Upon arrival in outer space, an astronaut's central blood volume increases. This leads to increased: A. pressure natriuresis B. renin, angiotensin, and aldosterone C. atrial natriuretic peptide D. sympathetic nervous system activity E. A and C are correct.

E

20. During exercise local metabolites are responsible for vasodilation in: A. skeletal muscle B. kidney C. heart D. lung E. A and C are correct.

E

21. A 10% hemorrhage and standing quietly for 30 minutes have similar effects on: A. pulse pressure B. capillary fluid shifts in leg capillaries C. central blood volume D. peripheral blood volume E. A and C are correct.

E

24. Activation of parasympathetic nerves causes: A. dilation of arterioles in the skin B. dilation of the bronchi C. dilation of arterioles in skeletal muscle blood vessels D. generalized sweating E. None of the above is correct.

E

4. A 42-year-old woman comes to the physician for a routine examination. She says that she has felt well except for occasional episodes of constipation, abdominal discomfort, and mild fatigue. She was treated for a renal calculus 10 years ago and was told she had a "lazy gallbladder." Her pulse is 82/min, and blood pressure is 150/80 mm Hg. Physical examination shows no other abnormalities. Laboratory studies show: Erythrocyte count 3 million/mm3 Serum K + 4.5 mEq/L Cl- 107 mEq/L Ca2+ 12 mg/dL Phosphorus 2.2 mg/dL Alkaline phosphatase 95 U/L The most likely cause of this patient's condition is a small, well-defined nodule in which of the following locations? (A) Adrenal gland (B) Anterior pituitary gland (C) Gallbladder (D) Kidney (E) Parathyroid gland (F) Thymus

E

51. A 31-year-old woman with type 2 diabetes mellitus comes to the physician because of an oozing, foul-smelling wound on her foot for 2 days. Physical examination shows a 4-cm, necrotizing wound with a purplish black discoloration over the heel. Crepitant bullae producing profuse amounts of serous drainage are seen. A Gram stain of a tissue biopsy specimen shows gram-positive rods. The causal organism most likely produces which of the following virulence factors? (A) Endotoxin (B) Fimbriae (C) Pneumolysin (D) Polysaccharide capsule (E) α-Toxin

E

76. A 73-year-old woman comes to the physician because of a 2-month history of diffuse weakness and tingling of her arms and legs. Neurologic examination shows weakness of the extensor and flexor muscles of the lower extremities. Knee and ankle deep tendon reflexes are exaggerated. Sensation to vibration and position is decreased in all extremities, but the decrease is more prominent in the lower extremities than in the upper extremities. This patient most likely has a deficiency of which of the following vitamins? (A) Niacin (B) Vitamin B1 (thiamine) (C) Vitamin B2 (riboflavin) (D) Vitamin B6 (pyridoxine) (E) Vitamin B12 (cyanocobalamin)

E

86. A 55-year-old man who is a business executive is admitted to the hospital for evaluation of abdominal pain. He is polite to the physician but berates the nurses and other staff. The patient's wife and two of his three adult children arrive for a visit. The patient says with disgust that the missing child is and always has been worthless. Which of the following is the most likely explanation for this patient's behavior? (A) Countertransference (B) Projection (C) Projective identification (D) Reaction formation (E) Splitting

E

89. A 17-year-old boy comes to the emergency department because of severe thirst and weakness and a 4-kg (8.8-lb) weight loss over the past 36 hours. He began having voluminous painless watery diarrhea on the airplane while returning from a trip to Thailand 36 hours ago. He has not vomited. While supine, pulse is 110/min and blood pressure is 110/60 mm Hg. While standing, pulse is 170/min and blood pressure is 70/40 mm Hg. His abdomen is nontender and bowel sounds are increased. Which of the following treatments is most appropriate at this time? (A) Ciprofloxacin (B) Doxycycline (C) Exploratory laparotomy (D) Potassium chloride (E) Rehydration (F) Trimethoprim-sulfamethoxazole

E

93. A 49-year-old woman is found to have mild proteinuria and pyuria on routine screening. Urine specific gravity is 1.000. Culture of the urine grows no organisms. She has taken large doses of combination over-the-counter analgesic preparations for 10 years due to a low back injury. Which of the following renal abnormalities is most likely in this patient? (A) Acquired cystic disease (B) Acute glomerulonephritis (C) Hyperplastic arteriolitis (D) Nephrolithiasis (E) Papillary necrosis

E

96. A 35-year-old man comes to the physician because of pain and swelling of his right arm where he scraped it on a tree branch 2 days ago. His temperature is 38.3°C (101°F). Examination of the right forearm shows edema around a fluctuant erythematous lesion at the site of trauma. The area is extremely tender to palpation. Which of the following is most likely the primary mechanism of the development of edema in this patient? (A) Degranulation of eosinophils (B) Disruption of vascular basement membranes (C) Increased hydrostatic pressure (D) Release of thromboxane (E) Separation of endothelial junctions

E

97. A 72-year-old man collapses while playing golf. He has a 5-year history of angina and type 2 diabetes mellitus. Paramedics arrive in 10 minutes. Examination shows no respirations or blood pressure; an ECG shows asystole. Cardiopulmonary resuscitation is attempted for 10 minutes without success. Which of the following is the most likely cause of death in this patient? (A) Cardiac tamponade (B) Embolus to the right middle cerebral artery (C) Necrosis of the myocardium (D) Rupture of the papillary muscle (E) Ventricular fibrillation

E

98. A 60-year-old woman with reflux esophagitis comes to the physician for a follow-up examination. She has been undergoing protonpump inhibitor therapy for the past month and shows clinical improvement. Upper endoscopy is done. A biopsy specimen of the stomach is most likely to show which of the following? (A) Acute inflammation (B) Gastric atrophy (C) Intramucosal smooth muscle (D) Lymphoid nodules (E) Parietal cell hyperplasia

E

87. A 14-year-old girl is brought to the physician by her mother because of a 2-month history of heavy vaginal bleeding during menstrual periods. She has had episodes of excessive periodontal bleeding while brushing her teeth and easy bruising for 6 years. She also had an episode of extended bleeding after a tooth extraction 4 years ago. Her mother and brother have had similar symptoms. Physical examination shows patchy ecchymoses over the upper and lower extremities. Laboratory studies show: Platelet count 234,000/mm3 Bleeding time 17 min Prothrombin time 12 sec (INR=1) Partial thromboplastin time 46 sec Which of the following is the most likely diagnosis? (A) Factor VII (proconvertin) deficiency (B) Factor X (Stuart factor) deficiency (C) Factor XII (Hageman factor) deficiency (D) Hemophilia A (E) Vitamin K deficiency (F) von Willebrand disease

F

57. A 64-year-old man with non-Hodgkin lymphoma comes to the physician because of a 3-week history of progressive numbness in his hands and feet and weakness in his legs when he stands. He received his third course of chemotherapy 4 weeks ago. Physical examination shows areflexia. Which of the following drugs is the most likely cause of these adverse effects? (A) Bleomycin (B) Cyclophosphamide (C) Cytarabine (D) Doxorubicin (E) Fluorouracil (F) Methotrexate (G) Vincristine

G

74. A 52-year-old man with recently diagnosed type 2 diabetes mellitus comes to the physician for a follow-up examination. Physical examination shows no abnormalities. Laboratory studies show an increased hemoglobin A1c despite patient compliance with diet and exercise recommendations. Treatment with a sulfonylurea is started. Which of the following is most likely to occur in this patient? (A) Decreased entry of glucose into the muscle cells (B) Decreased production of glucose from the liver (C) Decreased secretion of insulin from the pancreas (D) Decreased speed of carbohydrate absorption from the intestines (E) Increased entry of glucose into the muscle cells (F) Increased production of glucose from the liver (G) Increased secretion of insulin from the pancreas (H) Increased speed of carbohydrate absorption from the intestines

G

What relationship w GLUT4 accounts for the decline in blood glucose levels of someone after injecting insulin?

GLUT4 has a low Km for glucose -this kinetic feature of this transporter reflects its ability to continue to bind and move glucose into the cell even at low blood glucose concentrations. This is in contrast to GLUT2 on the liver and pancreas. In a state of insulin shock, it is unlikely that GLUT2 is transporting any glucose into the pancreas or liver, but GLUT4 may still be transporting blood glucose into m. and adipose. If blood glucose numbers decrease far enough, GLUT4 receptors will also eventually cease transport.

Which AA is converted to glutamate and then to the TCA cycle intermediate, alpha ketoglutarate when degraded?

Histidine

One group of Hemoglobins that results from single-point mutations all have the characteristic that the iron atom in the Hb tends to be more easily oxidized to the Fe3+ oxidation state, forming methemoglobin (the M type hemoglobin). Considering the 3D structure of Hb, where do you think these mutations might occur? No homozygous M-type hemoglobins are known. Why?

M-type hemoglobins: substitutions in or near the heme pocket which affect the heme protein bond --lots at either the distal or proximal histidine residue -integrity of the heme-protein pocket is essential for maintaining the iron atom in the reduced oxygenation state --free iron itself is relatively susceptible to oxidation by O2 --Hb and Mb are unusual in that they allow binding of the molecular oxygen without concomitant oxidation --the relatively hydrophobic nature of the heme pocket is imp for this, and anything that disrupts it would be detrimental to keeping the iron in the reduced oxidation state -heterozygosity tolerated bc half of the hemoglobin molecules are normal and can deliver at least some oxygen. Presumably, a homozygous indiv. would not be able to transport enough O2 to survive

A 44 yo moderately dehyrated man was admitted w a 2 day hx of acute severe diarrhea. Electrolyte results: Na+ 134, K+ 2.9, Cl- 108, HCO3- 16, BUN 31, Cr 1.5 ABG: pH 7.31 HCO3- 16 mEg/dl LOW LOW pCO2 33 mmHg pO2 93 mmHg What is the acid base disorder?

Metabolic acidosis

Which essential AA is needed to make cysteine?

Methionine (donates the sulfur, the Cs come from glucose)

Gout is caused by excessive uric acid levels in the blood and tissues. To determine whether a person with gout has developed this problem bc of overproduction of purine nucleotides or bc of a decreased ability to excrete uric acid, an oral dose of 15N labeled AA is sometimes used. Which AA would be most appropriate to use for this purpose?

The entire glycine molecule is incorporated into the precursor of the purine nucleotides. The nitrogen of this glycine also appears in uric acid, the product of purine degradation. 15N labeled glycine could be used, therefore, to determine whether purines are being overproduced.

As AAs are degraded, what are their Cs converted to?

1. CO2 2. compounds that produce glucose in the liver (pyruvate and the TCA cycle intermidiates, alpha ketoglutarate, succinyl coA, fumarate, oxaloacetate) 3. ketone bodies or their precursors (acetoacetate and acetyl coA)

How many amino acids can be synthesized in the body?

11

What % of blood is in the venous system at any given time?

75%

How many amino acids are essential and must be taken in from the diet?

9

17. Cerebral arterioles dilate in response to increased: A. arterial blood PCO2 B. brain interstitial pH C. arterial pressure D. sympathetic nerve activity E. All of the above are correct.

A

37. A 52-year-old man comes to the emergency department because he has had vomiting, nausea, and abdominal pain for the past 12 hours. He says he attempted suicide 3 days ago by "taking everything in the medicine cabinet." He was stuporous for approximately 12 hours after the overdose but felt better the following day. At this time, he has jaundice and pain in the right upper quadrant. Which of the following drugs is most likely to have caused the pain, vomiting, and jaundice? (A) Acetaminophen (B) Aspirin (C) Cimetidine (D) Diphenhydramine (E) Triazolam

A

5. During a cardiac cycle (beginning with atrial systole), the first heart sound: A. is associated with closure of the mitral valve B. is associated with closure of the aortic valve C. occurs before the QRS complex D. occurs after isovolumetric relaxation E. A and C are correct.

A

What is the B6 dependent enzyme involved in porphyrin synthesis?

ALA synthase

What could cause a diminished S1?

AV block, high diastolic filling pressures (heart failure, stiff ventricle)

Which form of Hb is always present, in fetus and adult?

Alpha subunit

39. A 2-year-old girl is brought to the emergency department because of pain in her right forearm after a fall 1 hour ago. She has a history of fractures of the left femur and right tibia. Physical examination shows blue sclerae. There is tenderness to palpation over the distal right radius. A mutation in which of the following genes is the most likely cause of the recurrent fractures in this patient? (A) Calcitonin (B) Collagen, type I (C) 1α-Hydroxylase (D) Parathyroid hormone (E) Vitamin D receptor

B

Propionic acid accumulation from AA degradation will result from a deficiency of which of the following vitamins? A. Vit. B6 B. Biotin C. Folic Acid D. Vit. B12 E. Vit. B1 F. Vit. B2

B. Proprionic acid is derived from an accumulation of propionyl-CoA. The normal pathway for the degradation of propionyl-CoA is, first, a biotin-dependent carboxylation to D-methylmalonyl-CoA, racemization to L-methylmalonyl-coA, and then, the B12 dependent rearrangement to succinyl CoA

If more CO2 is added to the blood, what effect does that have on pH?

Blood would be more ACIDIC

107. A 48-year-old man with AIDS has chorioretinitis with flame-shaped hemorrhages and large white patches. Three weeks after starting therapy, his vision is improved, but he has granulocytopenia. Which of the following antiviral agents is most likely responsible for these findings? (A) Amantadine (B) Foscarnet (C) Ganciclovir (D) Lamivudine (3TC) (E) Ribavirin

C

4. Which of the following occurs during ventricular isovolumetric contraction? A. opening of the mitral valve B. the second heart sound C. aortic pressure reaches its lowest value of the cardiac cycle D. ventricular volume reaches its lowest value of the cardiac cycle E. aortic flow reaches its highest value of the cardiac cycle

C

40. A 55-year-old woman with small cell carcinoma of the lung is admitted to the hospital to undergo chemotherapy. Six days after treatment is started, she develops a temperature of 38°C (100.4°F). Physical examination shows no other abnormalities. Laboratory studies show a leukocyte count of 100/mm3 (5% segmented neutrophils and 95% lymphocytes). Which of the following is the most appropriate pharmacotherapy to increase this patient's leukocyte count? (A) Darbepoetin (B) Dexamethasone (C) Filgrastim (D) Interferon alfa (E) Interleukin-2 (IL-2) (F) Leucovorin

C

43. A 46-year-old woman receives a non-Tlymphocyte-depleted, allogeneic bone marrow transplant from a matched, unrelated donor. Immunosuppressive therapy with cyclosporine is started. One month later, she has fever. Cytolytic destruction of the skin, gastrointestinal tract, and liver is seen, with associated dermatitis, enteritis, and hepatitis. Which of the following best explains these findings? (A) C3b deposition (B) Cytomegalovirus infection (C) Graft-versus-host disease (D) Tolerance induction (E) Type I (immediate) hypersensitivity

C

46. Vascular control is studied in an intact hind extremity of an anesthetized experimental animal. After a normal control period, the blood flow to the extremity is completely occluded for 1 minute. When the occlusion is released, blood flow increases abruptly and exceeds the control value for several minutes (reactive hyperemia). After an appropriate recovery period, the procedure is repeated and the extremity is actively exercised during the occlusion period. Which of the following best describes the reactive hyperemia after the second occlusion compared with that after the first occlusion? (A) Abolished (B) Decreased but not abolished (C) Increased (D) Unchanged

C

60. A 30-year-old man is brought to the emergency department 30 minutes after being stung by several wasps. He is confused and has difficulty breathing. His temperature is 38°C (100.4°F), pulse is 122/min, respirations are 34/min, and blood pressure is 80/40 mm Hg. Physical examination shows dry skin and decreased capillary refill. There are multiple erythematous, inflamed marks on the back and 1+ pitting edema of the ankles. In addition to the administration of 0.9% saline, the most appropriate next step in management is administration of which of the following? (A) Atropine (B) Captopril (C) Epinephrine (D) Losartan (E) Methacholine (F) Whole blood

C

83. A 45-year-old woman has a 6-month history of progressive shortness of breath on exertion. She does not smoke. Pulmonary function findings are shown (values are given as % of predicted normal): Vital capacity 60 Forced expiratory volume in 1 second (FEV1) 70 Diffusing capacity for carbon monoxide 50 Maximum voluntary ventilation 60 Which of the following most likely explains her limited ability to increase ventilation? (A) Airway obstruction (B) Decreased activation of pulmonary juxtacapillary (J) receptors (C) Decreased lung compliance (D) Depression of central chemoreceptors (E) Depression of peripheral chemoreceptors

C

91. A 66-year-old man has become increasingly short-tempered with his wife. He has diarrhea, weight loss, and weakness in the proximal muscles. He has atrial fibrillation and tachycardia. Which of the following is the most likely diagnosis? (A) Congestive heart failure (B) Cushing syndrome (C) Hyperthyroidism (D) Mitral valve prolapse (E) Pheochromocytoma

C

What is the most oxidized form of C in the body?

CO2

79. A 16-year-old girl has hirsutism, deepening of the voice, and cessation of menses. She swims in competitions. Which of the following drugs is most likely to have caused these findings? (A) Ethinyl estradiol (B) Leuprolide (C) Medroxyprogesterone (D) Nandrolone (E) Tamoxifen

D

Which parameter best indexes a diabetic's level of glucose control over the past 3 mos?

HbA1c

Despite admitting to alcohol use that constitutes abuse for 5 yrs, Jean Ann Tonich is now presenting w sx associated w anemia. What is the most likely reason for this?

Her endogenous stores of vit. B12 have maintained sufficient substrate levels required for cell division

Which AAs are strictly ketogenic?

Lysine and leucine

Degradative pathways of AAs generate what product?

NADH, an electron source for oxidative phosphorylation

Splenomegaly is an indicator of what?

RBC abnormalities, causing premature destruction of RBCs or Hemolysis

A doctor is reviewing a pt's arterial blood gas (ABG) values. Which condition would apply? pH 7.49 bicarb ion 24 mEg/dl PaCO2 3 1mmHg PaO2 52 mmHg

Respiratory alkalosis

Which AAs produce pyruvate when degraded?

Serine, Alanine, Cysteine (made from intermediates of glycolysis) and threonine, glycine, tryptophan

When the C skeleton of alanine is derived from glucose, the efflux of alanine from skeletal m. and its uptake by liver provide no net transfer of AA C to the liver for gluconeogenesis. However, some of the alanine C is derived from sources other than glucose. Which AAs can provide C for alanine formation?

Some of the alanine from skeletal m is derived directly from protein degradation. The C skeletons of valine, isoleucine, aspartate, and glutamate, which are converted to malate and oxaloacetate in the TCA cycle, can be converted to pyruvate and subsequently transaminated to alanine. The extent to which these AAs contribute C to alanine efflux differs btwn diff types of muscles in the human. These AAs also may contribute to alanine efflux from the gut.

Untreated hemolytic anemias can culminate in certain facies, such as frontal bossing and abnormal bone expansion. Why?

The anemia results in secretion of EPO in an effort to fix the problem, but due to the untreated disease, there is a futile expansion of bone marrow.

What causes sickle cell (HbS)?

a point mutation (E6V) in the beta-subunit of Hb

How does Jean Ann Tonich metabolize the excess alcohol she has consumed?

a primary alcohol is oxidized to an aldehyde and then further oxidized to a carboxylic acid while producing reduced coenzymes -Remember during metabolism, oxidation and reduction occur at the same time

What causes jaundice?

abnormal accumulations of bilirubin

Which would constitute an effective pH buffer? a.HCl and NaCl b. acetic acid and NaOH c. HCl and NaOH d. acetic acid and sodium acetate e. H2SO4 and SO4(2-)

acetic acid and sodium acetate

There are more known mutations documented in the Beta - chain of Hb than in the alpha-chain, and the gamma-chain variants seem to have serious consequences. why do you think this is so?

alpha-chain substitutions not only affect adult Hb but also fetal (2 alpha and 2 gamma chains) --these substitutions would cause problems in early development that would lead to spontaneous abortion -Beta-chain is not expressed to any great extent until after birth. These mutations do not show up during early development.

Where do RBCs get their energy?

anaerobic glycolysis - glucose

Which form of bilirubin is conjugated, or direct?

bilirubin-diglucuronide, conjugated by UDP, increasing the solubility and excretion

The 1st heart sound "lub" is caused by what?

closure of tricuspid and mitral valves at onset of ventricular systole

What type of murmur occurs after S2, but prior to S1?

diastolic

What are the 3 layers of the heart?

endocardium myocardium epicardium (visceral pericardium)

Which layer of blood vessels has an antithrombogenic role?

endothelium

What could cause an accentuated S1?

exercise, symp stimulation, mitral stenosis

What is the fate of glucose in RBCs?

lactate, in all states (fed, starving, fasting...)

What are the types of blood flow?

laminar and turbulent

What type of anemia could be caused by a vit. B12 deficiency?

macrocytic, normochromic

What type of anemia could be caused by folic acid deficiency?

macrocytic, normochromic

What type of anemia could be caused by iron deficiency?

microcytic, hypochromic

What type of anemia could be caused by lead poisoning?

microcytic, hypochromic

What type of anemia would be associated with vit. B6 deficiency?

microcytic, hypochromic anemia

Which condition/action might mimic the effects of injecting excess insulin in a type 1 diabetic?

prolonged exercise -increases insulin sensitivity and lowers blood glucose. Partly due to consumption of ATP from active mm. which activates various signaling cascades. Muscle activity will promote GLUT4 deposition of the cell surface independent of insulin signaling. Most diabetics that begin exercise routines or increase training will have to adjust insulin accordingly.

How would you treat an acidodic patient?

push bicarb (weak base) to neutralize the acid

What type of capillaries are characterized by a torturous path with unusually wide lumen?

sinusoidal (discontinuous)

What is the law of LaPlace?

tension as a function of radius

What happens to tension if the radius of the heart is increased, like in CHF?

tension increases

Which form of bilirubin is unconjugated, or indirect?

the bilirubin-albumin complex

During degradation of AAs, the energy generating pathway can involve which steps?

-direct oxidation -oxidation in TCA cycle -conversion to glucose then oxidation -conversion to ketone bodies then oxidation

Which AAs are made from glucose thru components of the glycolytic pathway?

-serine -glycine -cysteine -alanine

25. A technician wants to determine whether cytomegalovirus (CMV) DNA is present in the blood of a bone marrow transplant recipient. DNA purified from the leukocytes of the patient is reacted in a mixture containing oligonucleotides specific for CMV DNA, thermostable DNA polymerase, and nucleotides. Repetitive cycles of heating and cooling are performed, and the reaction product is detected by gel electrophoresis. The technician most likely used which of the following laboratory procedures on this patient's blood? (A) Northern blotting (B) Polymerase chain reaction (C) Reverse transcription (D) Southern blotting (E) Western blotting

B

29. A 7-month-old infant is brought to the physician's office because of poor weight gain despite large food intake. He has had two episodes of pneumonia and has frequent bulky stools. He coughs frequently. X-rays of the lungs show increased markings and hyperinflation. Trypsin is absent in a fresh stool sample, and the fat content is increased. Which of the following is the most likely cause of this infant's disorder? (A) Autoimmune disorder (B) Defective ion transport at epithelial surfaces (C) Disaccharidase deficiency (D) Inability to synthesize apolipoprotein B (E) Villous atrophy of the jejunum

B

3. A 4-year-old boy has delayed motor development and choreoathetosis. He had normal development at birth. He chews his fingers and lips, which has resulted in tissue loss. He has arthritis. Serum and urine uric acid concentrations are increased. Which of the following abnormalities is the most likely cause of these findings? (A) Adenine phosphoribosyltransferase deficiency (B) Hypoxanthine-guanine phosphoribosyltransferase deficiency (C) Increased cellular turnover of nucleic acids (D) Increased conversion of hypoxanthine to inosine monophosphate (E) Phosphoribosylpyrophosphate synthetase deficiency

B

34. A 6-year-old boy with glioblastoma has a recurrence of the tumor despite aggressive treatment. The physician discusses the patient's prognosis with his parents and recommends palliative care. The parents ask how they should talk with their son about his prognosis and possible death. The physician advises that the parents should be honest and follow the patient's lead during the conversation. This patient most likely has which of the following concepts of death? (A) Being asleep (B) Being final (C) Being a long journey (D) Being a temporary separation from his parents (E) No understanding of death

B

35. A 42-year-old man comes to the physician for a follow-up examination 1 week after he passed a renal calculus. X-ray crystallographic analysis of the calculus showed calcium as the primary cation. Physical examination today shows no abnormalities. A 24-hour collection of urine shows increased calcium excretion. Which of the following is the most appropriate pharmacotherapy? (A) Carbonic anhydrase inhibitor (B) Na+ -Cl− symport inhibitor (C) Na+ -K + -2Cl− symport inhibitor (D) Osmotic diuretic (E) Renal epithelial sodium channel inhibitor

B

11. A 15-year-old girl is brought to the physician because of a 3-week history of excessive thirst and voiding excessive amounts of urine. She shows no signs of kidney damage, and she is not taking any medications. Physical examination shows no abnormalities. She undergoes an 8- hour water deprivation test. She is also given 5 units of ADH (vasopressin), subcutaneously. Under both conditions, she continues to produce large volumes of dilute urine. Her symptoms are most likely due to a relative lack of which of the following proteins from the apical membranes of collecting duct epithelial cells? (A) Aquaporin (B) Epithelial Na+ channel (C) Na+-K+ ATPase (D) Na+-K+-2Cl−cotransporter (E) Urea transporter

A

12. A placebo-controlled clinical trial is conducted to assess whether a new antihypertensive drug is more effective than standard therapy. A total of 5000 patients with essential hypertension are enrolled and randomly assigned to one of two groups: 2500 patients receive the new drug and 2500 patients receive placebo. If the alpha is set at 0.01 instead of 0.05, which of the following is the most likely result? (A) Significant findings can be reported with greater confidence (B) The study will have more power (C) There is a decreased likelihood of a Type II error (D) There is an increased likelihood of statistically significant findings (E) There is an increased likelihood of a Type I error

A

12. When cardiac output doubles during exercise: A. pulmonary vascular resistance falls by almost 50% B. pulmonary vascular resistance remains unchanged C. pulmonary blood flow increases about 1.2-fold (i.e. about 20%) D. pulmonary artery pressure increases almost 2-fold E. pulmonary artery pressure falls about 2-fold

A

13. A sexually active 23-year-old man with multiple sex partners has dysuria and a yellow urethral exudate. Gram stain of the exudate shows numerous neutrophils, many that contain intracellular gram-negative diplococci. He has had three similar episodes of urethritis over the past 2 years. Which of the following properties of the infecting organism best explains the reinfection? (A) Antigenic variation (B) Catalase (C) Inhibition of B-lymphocyte function (D) Inhibition of T-lymphocyte function (E) Polysaccharide capsule

A

15. A 33-year-old woman comes to the physician because of a 2-day history of mild nausea, increased urinary urgency and frequency, and constipation. She also has had a 4.5-kg (10-lb) weight loss during the past 2 weeks and a 3- week history of vaginal bleeding. Pelvic examination shows a nodular cervix with an irregular, friable posterior lip, and a rock-hard, irregular, immobile pelvic mass that extends across the pelvis. Examination of biopsy specimens from the cervix and anterior wall of the vagina show well-differentiated keratinizing squamous cell carcinoma. Which of the following best describes the pathogenesis of this patient's disease? (A) Inactivation of cellular p53 (B) Insertion of viral promotors adjacent to cellular growth factor genes (C) Specialized transduction (D) Transactivation of cellular growth factor genes by TAX (E) Translocation of CMYC to an Ig gene promoter

A

16. A 22-year-old man develops delusions, flattening of affect, catatonic behavior, hallucinations, and aphasia. Which of the following symptoms would be more likely to improve if this patient were treated with clozapine rather than with haloperidol? (A) Affective flattening and aphasia (B) Affective flattening and hallucinations (C) Aphasia and delusions (D) Catatonia and delusions (E) Hallucinations and catatonia

A

17. After infection with measles virus, a 6-year-old boy produces antibodies to all eight viral proteins. The next year he is again exposed to measles virus. Antibodies to which of the following viral proteins are most likely to be protective? (A) Hemagglutinin (B) Matrix (C) Nonstructural (D) Nucleocapsid (E) Polymerase

A

23. Which of the following types of drugs would be most likely to increase the heart rate in a normal, quietly resting subject? A. muscarinic cholinergic antagonist B. acetylcholinesterase inhibitor C. beta adrenergic antagonist D. alpha adrenergic agonist E. muscarinic cholinergic agonist

A

27. A 17-year-old girl has never had a menstrual period. Physical examination shows a normal female body habitus, normal breast development, and normal appearing external genitalia. She has no axillary or pubic hair. The patient refuses to have a pelvic or rectal examination. Which of the following is the most likely explanation for the clinical presentation? (A) Androgen insensitivity (B) Congenital adrenal hyperplasia (C) Ectodermal dysplasia (D) A psychiatric disorder (E) A sex chromosome mosaicism

A

13. In comparing the transport of lipid-soluble and water-soluble substances across the capillary wall, diffusion of water-soluble substances: A. occurs across a larger surface area B. uses gaps between endothelial cells C. is not dependent on a concentration gradient D. occurs mainly through the plasma membrane lipid bilayer of endothelial cells E. A and C are correct.

B

Similarities btwn carbamoyl phosphate synthetase I and carbamoyl phosphate synthetase II include which one of the following? A. carbon source B. intracellular location C. N source D. Regulation by N-acetyl glutamate E. regulation by UMP

A Both carbomoyl phosphate synthetase (CPS) I and II use CO2 as the C source in the production of carbamoyl phosphate. CPSI is located in the mitochondria, whereas CPSII is in the cytoplasm (thus, B is incorrect). CPSI can fix ammonia; CPSII requires glutamine as the N source (thus, C is incorrect). N-acetylglutamate activates CPSI; CPSII is activated by PRPP (thus, D is incorrect.) UMP inhibits CPSII, but has no effect on CPSI (thus, E is incorrect)

8. A 62-year-old woman comes to the physician because of low back pain for 1 week. Menopause occurred 10 years ago. Physical examination shows localized tenderness over the lumbar spine after movement. X-rays of the spine show a compression fracture of L1-2. A DEXA scan shows decreased bone mineral density. Serum calcium and phosphorus concentrations and serum alkaline phosphatase activity are within the reference ranges. A bisphosphonate drug is prescribed. The expected beneficial effect of this drug is most likely due to which of the following actions? (A) Decreased insulin-like growth factor-1 concentration (B) Decreased osteoclast activity (C) Decreased osteoprotegerin production (D) Increased 1,25- dihydroxycholecalciferol concentration (E) Increased osteoblast activity (F) Increased receptor activator of NF-κB ligand (RANKL) production

B

9. Hospital discharge of a 75-year-old man is delayed due to unavailability of a bed in a nursing home. He is bedridden and unable to attend to his personal needs. During a 3-day period, his pulse increases from 82/min to 125/min, and blood pressure decreases from 124/72 mm Hg to 100/55 mm Hg. Laboratory values include: Day 1 Day 3 Hemoglobin 16.4 g/dL 18.4 g/dL Serum Urea nitrogen 18 mg/dL 56 mg/dL Glucose 100 mg/dL 89 mg/dL Na+ 135 mEq/L 151 mEq/L Creatinine 1.1 mg/dL 1.2 mg/dL Which of the following is the most likely diagnosis? (A) Acute renal failure (B) Dehydration (C) Diabetic ketoacidosis (D) Gastrointestinal hemorrhage (E) Syndrome of inappropriate ADH (vasopressin)

B

Pyridoxal phosphate is required for which of the following rxn pathways or individual rxns? A. Phenylalanine --> tyrosine B. Methionine --> cysteine + alpha-ketobutyrate C. Propionyl-CoA --> succinyl-CoA D. Pyruvate --> acetyl CoA E. Glucose --> glycogen

B. The rxn pathway in which methionine goes to cysteine and alpha-ketobutyrate requires pyridoxal phosphate at 2 steps: the cystathionine beta-synthase rxn and the cystathionase rxn. Phenylalanine to Tyr requires BH4; propionyl-CoA to succinyl coA requires B12; pyruvate to acetyl-CoA requires thiamine pyrophosphate, lipoic acid, NAD, FAD, and coenzyme A; and glucose to glycogen does not require a cofactor.

How should vit B12 be administered to a pt w pernicious anemia?

Bc the problem in pernicious anemia is a lack of intrinsic factor, which results in an inability to absorb vit B12 from the GI tract, B12 cannot be administered orally to treat this condition. In the past, it was usually given by injection. An effective nasal spray containing B12 has recently been marketed, however, and its use precludes the need for lifelong injections of this vit.

Gout can result from a reduction in activity of which one of the following enzymes? A. glutamine phosphoribosyl amidotransferase B. Glucose 6-phosphatase C. Glucose-6-phosphate dehydrogenase D. PRPP synthetase E. Purine nucleoside phosphorylase

B A lack of glucose 6-phosphatase activity (von Gierke disease) leads to an accumulation of glucose 6-phosphate, which leads to an increase in ribose 5-phosphate levels, and then an increase in PRPP levels. As PRPP levels rise, purine synthesis is stimulated, leading to excessive levels of purines in the blood. The degradation of the extra purines leads to uric acid production and gout. A loss of either PRPP synthetase activity or glutamine phosphoribosyl aminotransferase activity would lead to reduced purine synthesis and hypouricemia (thus, A and D are incorrect). A lack of glucose-6-phosphate dehydrogenase would hinder ribose 5-phosphate production and thus would not lead to excessive purine synthesis. A lack of purine nucleoside phosphorylase would hinder the salvage pathway, leading to an accumulation of nucleosides. Purine nucleoside phosphorylase activity is required to synthesize uric acid, so in the absence of this enzyme, less uric acid would be produced (thus, E is incorrect).

26. A 6-year-old boy from rural Mississippi is brought to the physician by his mother because of a 6-month history of lethargy; he also has had a 4-kg (9-lb) weight loss during this period. The mother says her son used to be active, often playing outside without wearing his shoes. The patient is at the 10th percentile for height and 12th percentile for weight. He appears pale. Physical examination shows pale oral mucosa. Laboratory studies show iron deficiency anemia. Microscopic examination of the stool shows thin-shelled ova. Which of the following is the most appropriate pharmacotherapy for this patient? (A) Doxycycline (B) Ivermectin (C) Mebendazole (D) Mefloquine (E) Trimethoprim-sulfamethoxazole

C

Allopurinol can be used to treat gout bc of its ability to inhibit which one of the following rxns? A. AMP to XMP B. Xanthine to uric acid C. inosine to hypoxanthine D. IMP to XMP E. XMP to GMP

B. Allopurinol inhibits the conversion of both hypoxanthine to xanthine and xanthine to uric acid. This occurs bc both of those rxns are catalyzed by xanthine oxidase, the target of allopurinol. Answer A is incorrect bc AMP is not converted directly to XMP (AMP, when degraded, is deaminated to form IMP, which loses its phosphate to become inosine, which undergoes phosphorolysis to generate hypoxanthine and ribose 1-phosphate). Answer C is incorrect bc the inosine-to-hypoxanthine conversion, catalyzed by nucleoside phosphorylase, is not inhibited by allopurinol. Answer D is incorrect bc the conversion of hypoxanthine to xanthine occurs at the free base level, not at the nucleotide level. Answer E is incorrect bc GMP is converted first to guanosine (loss of phosphate), the guanosine is converted to guanine and ribose 1-phosphate, and the guanine is then converted to xanthine by guanase.

The regulation of ribonucleotide reductase is quite complex. Assuming that an enzyme deficiency leads to highly elevated levels of dGTP, what effect would you predict on the reduction of ribonucleotides to deoxyribonucleotides under these conditions? A. elevated levels of dCDP will be produced B. The formation of dADP will be favored C. AMP will begin to be reduced D. Reduced thioredoxin will become rate-limiting, thereby reducing the activity of ribonucleotide reductase E. Deoxy-GTP will bind to the overall activity site and inhibit the functioning of the enzyme

B. If dGTP were to accumulate in cells, the dGTP would bind to the substrate specificity site of ribonucleotide reductase and direct the synthesis of dADP. This would lead to elevations of dATP levels, which would inhibit the activity of ribonucleotide reductase. The inhibition of ribonucleotide reductase leads to a cessation of cell proliferation, as the supply of deoxyribonucleotides for DNA synthesis become limiting. Answer A is incorrect bc ATP would need to bind to the substrate specificity site to direct the synthesis of dCDP. That would not occur under these conditions of elevated dGTP levels. Answer C is incorrect bc the enzyme works only on diphosphates; AMP would never be a substrate for this enzyme. Answer D is incorrect bc the thioredoxin is always regenerated and does not become rate limiting for the reductase rxn. Answer E is incorrect bc dGTP does not bind to the activity site of the reductase; only ATP (activator) or dATP (inhibitor) is capable of binding to the activity site.

In what ways does liver metabolism after a high protein meal resemble liver metabolism in the fasting state?

Both of these dietary states are characterized by an elevation of glucagon. Glucagon stimulates AA transport into the liver, stimulates gluconeogenesis thru decreasing levels of fructose 2,6-bisphosphate, and induces the synthesis of enzymes in the urea cycle, the gluconeogenic pathway, and the pathways for degradation of some of the AAs.

1. A 25-year-old woman is brought to the emergency department 1 hour after she fainted. She has had mild intermittent vaginal bleeding, sometimes associated with lower abdominal pain, during the past 3 days. She has had severe cramping pain in the right lower abdomen for 12 hours. She has not had a menstrual period for 3 months; previously, menses occurred at regular 28-day intervals. Abdominal examination shows mild tenderness to palpation in the right lower quadrant. Bimanual pelvic examination shows a tender walnut-sized mass in the right parametrium. Which of the following is the most likely diagnosis? (A) Appendicitis (B) Cancer of the ovary (C) Ectopic pregnancy (D) Endometriosis (E) Ovarian cyst (F) Placenta previa

C

1. Blood in the pulmonary artery: A. has an oxygen content which is approximately twice that of blood in the aorta B. has a carbon dioxide content which is approximately half that in the pulmonary veins C. flows at a rate equal to the flow rate from the left ventricle into the aorta D. passes through the tricuspid valve immediately before entering the left ventricle E. is not considered to be part of the central blood volume

C

23. A 38-year-old man who recently immigrated to the USA comes to the physician because of a 1-month history of cough and a 4.5-kg (10-lb) weight loss. Physical examination shows no abnormalities. A chest x-ray shows a right upper lobe infiltrate. One of three sputum samples is positive for acid-fast bacilli. Treatment with isoniazid, rifampin, ethambutol, and pyrazinamide is started. Which of the following should be added to the medication regimen to prevent neurologic toxicity in this patient? (A) Folic acid (B) Nicotinic acid (C) Vitamin B6 (pyridoxine) (D) Vitamin B12 (cyanocobalamin) (E) Vitamin C

C

24. A 12-year-old girl is brought to the emergency department by her parents because of a 3-day history of fever and a 12-hour history of lethargy. Her parents say that she has been sleeping most of the day and has been unresponsive when awake. Her temperature is 39.2°C (102.6°F). Physical examination shows numerous petechial hemorrhages and nuchal rigidity. A lumbar puncture yields cloudy cerebrospinal fluid (CSF) that clots in the collection tube. Microscopic examination of the CSF shows numerous segmented neutrophils, and a Gram stain shows gramnegative diplococci. Which of the following is the most likely causal organism? (A) Haemophilus influenzae (B) Mycoplasma pneumoniae (C) Neisseria meningitidis (D) Salmonella typhi (E) Streptococcus pneumoniae

C

25. The effects of cocaine on the heart could be most effectively counteracted by administering: A. an alpha1 adrenergic agonist B. an alpha1 adrenergic antagonist C. a beta1 adrenergic antagonist D. a muscarinic (M2) cholinergic antagonist E. a drug that blocks the reuptake of norepinephrine into adrenergic neuron terminals

C

28. A 40-year-old woman comes to the physician because of a 6-month history of increased facial hair growth. Her last menstrual period was 4 months ago. She is 165 cm (5 ft 5 in) tall and weighs 70 kg (154 lb); BMI is 26 kg/m2 . Her pulse is 80/min, and blood pressure is 130/82 mm Hg. Physical examination shows temporal balding and coarse dark hair on the upper lip and chin. Pelvic examination shows clitoral enlargement. Her serum testosterone concentration is increased. Serum concentrations of androstenedione, dehydroepiandrosterone, and urinary 17-ketosteroids are within the reference ranges. Ultrasonography of the pelvis shows a 12-cm ovarian mass. Which of the following best describes this mass? (A) Granulosa tumor (B) Ovarian carcinoid (C) Sertoli-Leydig tumor (D) Teratoma (E) Thecoma

C

33. A 15-year-old girl comes to the physician because of a 3-month history of acne. Breast and pubic hair development began at the age of 12 years. Menarche occurred at the age of 14 years. Physical examination shows scattered open and closed comedones over the cheeks and forehead. Breast and pubic hair development are Tanner stage 5. Which of the following is the most likely underlying cause of this patient's acne? (A) Decreased parasympathetic stimulation to the sebaceous glands (B) Increased estrogen stimulation of the sebaceous glands (C) Increased responsiveness of the sebaceous glands to folliclestimulating hormone (D) Increased sympathetic stimulation to the sebaceous glands (E) Stimulation of the sebaceous glands by androgens

E

A dietary vit B12 deficiency can result from which of the following? A. excessive intrinsic factor production by the gastric parietal cells B. eating a diet that is high in animal protein C. pancreatic insufficiency D. increased absorption of folic acid E. inability to conjugate the vitamin with glutamic acid

C. An inability to hydrolyze dietary protein and release bound B12 will lead to a deficiency. Pancreatic insufficiency will result in reduction of proteases in the pancreas and reduced ability to release B12 from protein. Both R-binders and intrinsic factor are required for appropriate B12 absorption; animal protein contains high levels of B12, and glutamic acid is conjugated to folic acid, not to B12. Folic acid absorption does not affect B12 absorption.

A folic acid deficiency would interfere w the synthesis of which of the following AAs from the indicated precursors? A. Aspartate from oxaloacetate and glutamate B. Glutamate from glucose and ammonia C. Glycine from glucose and alanine D. Proline from glutamate E. Serine from glucose and alanine

C. Folic acid is required for the conversion of serine to glycine and the serine is produced from 3-phosphoglycerate and alanine. Folic acid is not needed to synthesize aspartate (from oxaloacetate by transamination), glutamate (from alpha-ketoglutarate by transamination), proline (from glutamate by a series of steps that do not require folic acid), or serine (from 3-phosphoglycerate, with no 1-C metabolism needed)

A newborn infant has elevated levels of phenylalanine and phenylpyruvate in her blood. Which of the following enzymes might be deficient in this baby? A. Phenylalanine dehydrogenase B. Phenylalanine oxidase C. Dihydropteridine reductase D. Tyrosine hydroxylase E. Tetrahydrofolate synthase

C. The classical form of PKU, a deficiency of phenylalanine hydroxylase, results in elevations of phenylalanine and phenylpyruvate. However, this enzyme is not a choice. In the nonclassical variant of PKU, there is a problem in either synthesizing or regenerating BH4. The enzyme that converts BH2 to BH4 is dihydropteridine reductase.

Lesch-Nyhan syndrome is due to an inability to catalyze which of the following rxns? A. Adenine to AMP B. Adenosine to AMP C. Guanine to GMP D> Guanosine to GMP E> Thymine to TMP F. Thymidine to TMP

C. The enzyme defect in Lesch-Nyhan syndrome involves hypoxanthine-guanine phosphoribosyltransferase (HGPRT). This enzyme converts the free base to a nucleotide - specifically, guanine to GMP and hypoxanthine to IMP. Adenine phosphoribosyltransferase (APRT) converts adenine to AMP (thus, A is incorrect). Adenosine kinase converts adenosine to AMP (thus B is incorrect). There are no enzymes to convert guanosine to GMP or thymine to TMP (so D and E are incorrect.) Pyrimidine nucleoside phosphorylase will convert thymine to thymidine but not to the nucleotide level. Thymidine kinase converts thymidine to TMP (so F is incorrect).

2. Compared to Purkinje cells: A. ventricular cells have a faster rate of depolarization during diastole (phase 4) B. sinoatrial (SA) nodal cells depend more on voltage-gated Na+ channels during the upstroke of the action potential (phase 0) C. atrial cells have more open Ca++ channels during the plateau of the action potential (phase 2) D. atrioventricular (AV) nodal cells have a slower rate of depolarization during the upstroke of the action potential (phase 0) E. atrioventricular (AV) nodal cells have a slower rate of depolarization during diastole (phase 4)

D

31. A new blood test to detect prostate cancer is evaluated in 300 male volunteers. A needle biopsy of the prostate gland is done on all men with serum prostate-specific antigen concentrations greater than 5 ng/mL (N<4). One hundred men undergo biopsy procedures; 90 are found to have prostate cancer, and five are found to have chronic prostatitis. Which of the following is necessary to calculate the sensitivity of this test? (A) Incidence of chronic prostatitis in the general population (B) Number of men with test results greater than 5 ng/mL and a normal biopsy specimen (C) Prevalence of chronic prostatitis in the general population (D) Prostate biopsies of men with test results equal to or below 5 ng/mL

D

9. Stimulation of sympathetic nerves increases cardiac output by: A. increasing heart rate B. increasing ventricular force of contraction C. increasing the duration of systole D. Only A and B are correct. E. A, B, and C are correct.

D

A 32-year-old woman with type 1 diabetes mellitus has had progressive renal failure over the past 2 years. She has not yet started dialysis. Examination shows no abnormalities. Her hemoglobin concentration is 9 g/dL, hematocrit is 28%, and mean corpuscular volume is 94 μm3 . A blood smear shows normochromic, normocytic cells. Which of the following is the most likely cause? (A) Acute blood loss (B) Chronic lymphocytic leukemia (C) Erythrocyte enzyme deficiency (D) Erythropoietin deficiency (E) Immunohemolysis (F) Microangiopathic hemolysis (G) Polycythemia vera (H) Sickle cell disease (I) Sideroblastic anemia (J) β-Thalassemia trait

D

Which of the following rxns requires N5, N10-methylene-FH4 as a C donor? A. Homocysteine to methionine B. Serine to glycine C. Betaine to dimethylglycine D. dUMP to dTMP E. the de novo biosynthesis of the purine ring

D the homocysteine to methionine rxn requires N5-methyl-FH4; serine to glycine requires free FH4 and generates N5, N10-methylene-FH4; betaine donates a methyl group to homocysteine to form methionine without the participation of FH4; and the purine ring requires N10-formyl-FH4 in its biosynthesis

30. A 50-year-old man has headache, vertigo, and generalized pruritus. He has the recent onset of angina pectoris. His hematocrit is 65%, leukocyte count is 12,000/mm3 , and erythrocyte mass is increased. Erythropoietin concentration is decreased. Which of the following is the most likely diagnosis? (A) Glucose-6-phosphate dehydrogenase deficiency (B) Hemochromatosis (C) Immune thrombocytopenic purpura (D) Pernicious anemia (E) Polycythemia vera (F) Pyruvate kinase deficiency (G) Secondary polycythemia

E

10. Mean arterial pressure can be closely approximated by: A. (cardiac output) times (systemic vascular resistance) B. (systemic vascular resistance) divided by (cardiac output) C. (one-third of pulse pressure) plus (diastolic pressure) D. (systolic pressure plus diastolic pressure) divided by 3 E. A and C are correct.

E

19. A 52-year-old woman comes to the physician because of a 2-day history of fever and left flank pain. She has been treated for multiple episodes of pyelonephritis during the past 3 years. Her temperature is 37.8°C (100.1°F). Physical examination shows left flank tenderness. Urinalysis shows 12-18 WBC/hpf with occasional lymphocytes and mononuclear cells with features of macrophages. Cultures of urine grow 80,000 colonies/mL of Proteus mirabilis. An x-ray of the abdomen shows a 3-cm mass in the lower pole of the left kidney. Gross examination of the mass after it has been resected shows that it is yellow, 3.2-cm in diameter, and centrally but not marginally necrotic. Histologic examination of the mass shows a predominance of epithelioid cells with partially clear and granular-tofoamy cytoplasm. Nuclei are eccentric, normochromic, symmetric, and without significant pleomorphism. Scattered lymphocytes and plasma cells are intermixed. Which of the following is the most likely diagnosis? (A) Acute pyelonephritis (B) Malacoplakia (C) Renal cell carcinoma, clear cell type, intermediate grade (D) Renal cell carcinoma, granular cell type (E) Xanthogranulomatous pyelonephritis

E

2. A 12-year-old girl with a 1-year history of systemic lupus erythematosus is brought to the physician for a routine follow-up examination. During the past year she has done well with the exception of occasional mild frontal headaches, fatigue, and arthralgias; the results of regular laboratory evaluations have been stable. She has missed 20 days of school in the past 4 months. Her parents express concern that sending her to school might cause her to be exposed to children with contagious diseases that might exacerbate their daughter's condition. The girl tells the physician that she misses seeing her many friends at school. Passive motion of the elbows, wrists, and knees produces mild discomfort. Physical examination shows no other abnormalities. Which of the following is the most likely cause of this patient's excessive school absences? (A) Lupus arthritis (B) Lupus cerebritis (C) Malingering (D) School phobia (E) Vulnerable child syndrome

E

An alternative method to methylate homocysteine to form methionine is which of the following? A. using glycine and FH4 as the methyl donor B. Using dimethylglycine as the methyl donor C. Using choline as the methyl donor D. Using sarcosine as the methyl donor E. Using betaine as the methyl donor

E. Choline, derived from phosphatidylcholine, is converted to betaine (trimethylglycine). Betaine can donate a methyl group to homocysteine to form methionine plus dimethylglycine. Sarcosine is N-methylglycine, which is formed when excess SAM methylates glycine, but is not used as a methyl donor in this rxn.

The degradation of AAs can be classified into families, which are named after the end product of the degradative pathway. Which of the following is such an end product? A. Citrate B. Glyceraldehyde 3-phosphate C. Fructose 6-phosphate D. Malate E. Succinyl CoA

E. The other end products are acetoacetate, acetyl-CoA, fumarate, oxaloacetate, alpha-ketoglutarate, and pyruvate

6. A 12-year-old girl with sickle cell disease has pain in her right arm. An x-ray of the right upper extremity shows bony lesions consistent with osteomyelitis. Which of the following is the most likely causal organism? (A) Clostridium septicum (B) Enterococcus faecalis (C) Listeria monocytogenes (D) Proteus mirabilis (E) Pseudomonas aeruginosa (F) Salmonella enteritidis (G) Serratia marcescens

F

A biopsy specimen from Homer Sistine's liver was sent to the hospital's biochem research lab for enzyme assays. Cystathionine beta-synthase activity was reported to be 7% of that found in normal liver. Homocystinuria is caused by deficiencies in the enzymes cystathionine beta-synthase and cystathionase as well as by deficiencies of methyltetrahydrofolate (CH3-FH4) or of methyl B12. The deficiencies of CH3-FH4 or of methyl-B12 are caused by either an inadequate dietary intake of folate or vit B12 or by defective enzymes involved in joining methyl groups to FH4, transferring methyl groups from FH4 to B12, or passing them from B12 to homocysteine to form methionine. Is Homer Sistine's homocystinuria caused by any of these problems?

Homer Sistine's methionine levels are elevated, and his vit B12 and folate levels are normal. Therefore, he does not have a deficiency of dietary folate or vit B12 or of the enzymes that transfer methyl groups from FH4 to homocysteine to form methionine. In these cases, homocysteine levels are elevated but methionine levels are low.

Homocysteine is oxidized to a disulfide - homocystine. To indicate that both forms are being considered, the term "homocyst(e)ine" is used Bc a colorimetric screening test for urinary homocystine was +, the dr ordered several biochemical studies on Homer Sistine's serum, which included tests for methionine, homocyst(e)ine (both free and protein bound), cystine, vit B12, and folate. The level of homocystine in a 24 hr urine collection also measured. Results: serum methionine level was 980 micromolars (reference <30), serum homocyst(e)ine (both free and protein bound) markedly elevated, cystine not detected in serum, serum B12 and folate levels normal. 24 hr urine homocystine level was elevated. Homer Sistine's dr concluded that Homer had homocystinuria caused by an enzyme deficiency. What was the rationale for this conclusion?

If the blood levels of methionine and homocysteine are very elevated and cystine is low, cystathionine beta-synthase could be defective, but a cystathionase deficiency is also a possibility. With a deficiency of either of these enzymes, cysteine could not be synthesized, and levels of homocysteine would rise. Homocysteine would be converted to methionine by rxns that require vit B12 and FH4 In addition, it would be oxidized to homocystine, which would appear in the urine. The levels of cysteine (measured as its oxidation product of cystine) would be low. A measurement of serum cystathionine levels would help to distinguish btwn a cystathionase or cystathionine beta-synthase deficiency.

What compounds form succinyl-CoA via propionyl-CoA and methyl-malonyl-CoA?

In addition to methionine, threonine, isoleucine, and valine the last 3 Cs at the w-end of odd-chain fatty acids form succinyl-CoA by this route.

In the fasting state, what happens with the degradation of AAs?

In the liver, C skeletons of AAs produce glucose, ketone bodies, and CO2

Which essential AA is needed to make Tyrosine?

Phenylalanine

Which AAs are synthesized from TCA cycle intermediates and are also degraded back into these intermediates?

aspartate asparagine glutamate glutamine proline arginine

What changes in hormone levels and fuel metabolism occur during an overnight fast?

The hormonal changes that occur during an overnight fast include a decrease of blood insulin levels and an increase of glucagon relative to levels after a high carb meal. Glucocorticoid levels also increase in the blood. These hormones coordinate the changes of fat, carb, and AA metabolism. Fatty acids are released from adipose triacylgylcerols and are used as the major fuel by heart, skeletal m, liver, and other tissues. The liver converts some of the fatty acids to ketone bodies. Liver glycogen stores are diminished and gluconeogenesis becomes the major support of blood glucose levels for glucose-dependent tissues. The major precursors of gluconeogenesis include AAs released from skeletal m, lactate, and glycerol.

What is the only tissue that has all the pathways of AA synthesis and degradation?

The liver

AAs whose C skeletons can be converted to a precursor of glucose are called what?

glucogenic

Which AAs form succinyl coA when degraded?

methionine, threonine, valine, isoleucine

Which AA forms fumarate when degraded?

phenylalanine (after conversion to Tyr)

Which AAs that are both glucogenic and ketogenic can form acetoacetate when degraded?

phenylalanine and tyrosine

In the fed state, what happens with the degradation of AAs?

the liver converts intermediates of AA metabolism to glycogen and triaglycerols

Which AAs that are both glucogenic and ketogenic can form acetyl coA when degraded?

tryptophan, isoleucine, threonine


Related study sets

Chapter 1 of Bontrager Textbook of Radiolographic Positioning (Self-test) Part B

View Set

Chapter 6: The Biomechanics of Skeletal Muscle

View Set

A&P I- chapter 7- The Integumentary System

View Set

CSC 415 Operating System Principles Chapter 2

View Set